Esercizi

1.
Data la base $\mathcal{B}=((0,1,i),(i,-2,0),(0,0,-1))$ dello spazio hermitiano $\mathbf{C}^3$, trovare attraverso il procedimento di ortogonalizzazione di Gram-Schimdt, una base ortonormale $\mathcal{C}$ per $\mathbf{C}^3$ rispetto al prodotto hermitiano standard.
Considerata la base canonica $\mathcal{E}$ di $\mathbf{C}^3$, che proprietà ha la matrice $M_{\mathcal{E,C}}(id_{\mathbf{C}^3})$? Perché?

Vuoi un aiuto?
Per vedere la soluzione totale, clicca , oppure clicca per visualizzare ogni singolo passo della soluzione.

Soluzione
I passo:
Cerchiamo la base $\mathcal{C}$:
$\mathbf{v}_{1}=(0,1,i)$;
$\vert\vert\mathbf{v}_{1}\vert\vert=1-i^2=2$, allora
$\quad \mathbf{w}_{1}=\frac{1}{\sqrt{2}}(0,1,i)$.
$\tilde{\mathbf{w}}_{2}=\mathbf{v}_{2}-(\mathbf{v}_{2}\cdot \mathbf{w}_{1})\mathbf{w}_{1}=$
$=(i,-2,0)+(0,1,i)=(i,-1,i)$;
$\vert\vert\tilde{\mathbf{w}}_{2}\vert\vert=i(-i)+1+i(-i)=3$, allora
$\quad \mathbf{w}_{2}=\frac{1}{\sqrt{3}}(i,-1,i)$.
$\tilde{\mathbf{w}}_{3}=\mathbf{v}_{3}-(\mathbf{v}_{3}\cdot \mathbf{w}_{1})\mathbf{w}_{1}-(\mathbf{v}_{3}\cdot \mathbf{w}_{2})\mathbf{w}_{2}=$
$=(0,0,-1)-\frac{i}{2}(0,1,i)-\frac{i}{3}(i,-1,i)=\frac{1}{6}(2,-i,-1)$;
$\vert\vert\tilde{\mathbf{w}}_{3}\vert\vert=\frac{1}{36}(4+(-i)i+1)=\frac{1}{6}$, allora
$\quad \mathbf{w}_{3}=\frac{1}{\sqrt{6}}(2,-i,-1)$.
II passo:  
Allora la base $\mathcal{C}=(\frac{1}{\sqrt{2}}(0,1,i),\frac{1}{\sqrt{3}}(i,-1,i),\frac{1}{\sqrt{6}}(2,-i,-1))$ è ortonormale e la matrice di passaggio fra le due basi diventa

\begin{displaymath}M_{\mathcal{E,C}}(id_{\mathbf{C}^3})=
\begin{array}({ccc})
...
...{2}} & \frac{i}{\sqrt{3}} & -\frac{1}{\sqrt{6}}
\end{array},
\end{displaymath}

che è una matrice unitaria, infatti svolgendo i calcoli si può verificare che $M_{\mathcal{E,C}}(id_{\mathbf{C}^3})(\overline{M_{\mathcal{E,C}}(id_{\mathbf{C}^3})})^{t}=I_{3}$.
Gli endomorfismi che portano basi ortonormali in basi ortonormali sono tutti endomorfismi unitari.